Search found 43 matches


I echo navalpike's words. That is an extremely bogus claim and you shouldn't be putting that much stock in it.

by jjk

Tue Sep 01, 2009 12:14 pm
Forum: I just Beat The GMAT!
Topic: Did I beat the GMAT?
Replies: 9
Views: 2370

Yep, that's just how it is. Your break time runs the duration of the time you're outside the testing room. Kinda sucks b/c when I took it, I would have liked a little time to sit down and recalibrate, but you just gotta roll with it.

by jjk

Mon Aug 31, 2009 7:21 pm
Forum: I just Beat The GMAT!
Topic: Testing center folly
Replies: 5
Views: 1996

Now THAT is how you demolish the GMAT. Awesome score.

by jjk

Fri Aug 21, 2009 9:25 am
Forum: I just Beat The GMAT!
Topic: Killed it - 790 (51 quant / 49 verbal)
Replies: 47
Views: 25328

From 640 to 730!!

My retake was today. My first attempt on 6/29 yielded a very disappointing 640 (40 Q, 38 V). Today's retake, though, was a different story. 730 (50 Q, 40 V) I owe a HUGE thank-you to the BTG community simply for posting challenging questions and excellent explanations, both of which helped refine my...

by jjk

Tue Aug 18, 2009 5:00 pm
Forum: I just Beat The GMAT!
Topic: From 640 to 730!!
Replies: 6
Views: 3691
by jjk

Tue Aug 18, 2009 7:45 am
Forum: Data Sufficiency
Topic: OG Diag Test Q doubt OA (0 also an integer)
Replies: 15
Views: 7017

If the product of 7 consecutive integers is not zero, is the product negative? 1.) The largest number is less than 7 2.) At least on of the numbers is negative A alone is sufficient.As From the question stem and A it is clear that the 7 numbers here are all negative and hence their product is negat...

by jjk

Mon Aug 17, 2009 6:52 pm
Forum: Data Sufficiency
Topic: product of 7 consecutive integers
Replies: 6
Views: 1898

B

You can test statement 1 by setting n equal to 9 or 18 and p equal to 3.

If n = 9 and p = 3, 9/3 and 9/9 yield integers.

If n = 18 and p = 3, 18/3 and 18/9 yield integers.

However, 18 is NOT a square of an integer. Statement 1 is thus insufficient.

by jjk

Mon Aug 17, 2009 6:47 pm
Forum: Data Sufficiency
Topic: Prime & Square
Replies: 29
Views: 13173

It's C When both statements are combined, T MUST BE POSITIVE. The left side of the equation is an absolute value, which is positive. Therefore, T unmodified must be positive. With |ST| = T, we know that S is 1 b/c the passage already states that S > 0. Therefore, S cannot be -1. We know that T unmod...

by jjk

Mon Aug 17, 2009 4:32 pm
Forum: Data Sufficiency
Topic: inequalities with mod ds
Replies: 14
Views: 2352

If n is a positive integer and r is the remainder when 4+7n divided by 3, what is the value of r? (1) n+1 is divisible by 3 (2) n>20 Will be A. If n+1 is divisible by 3 then n will always give a rem of 2. The problem does not ask you the value of r when n is divided by 3. It asks you the value of r...

by jjk

Mon Aug 17, 2009 4:14 pm
Forum: Data Sufficiency
Topic: Remainder R
Replies: 14
Views: 5785

Yes, Y is simply Z - X.

If we replace Y with Z - X in the equation, we get:

.10X + .02(16 - X) = .8

X = 6

by jjk

Mon Aug 17, 2009 3:48 pm
Forum: Data Sufficiency
Topic: A contractor
Replies: 10
Views: 2088

E Statement 1 -- INSUFFICIENT Although xy = 1, we do not know whether x and y are integers nor whether they are positive. x could be .25 and y could be 4, giving us varied answers. x could also be -.25 and y could be -4. If y is -4 and raised to an odd exponent, the value of x^2b * y^3b will definit...

by jjk

Mon Aug 17, 2009 3:17 pm
Forum: Data Sufficiency
Topic: DS with Exponents
Replies: 2
Views: 1253

Answer is A. Using a simple formula to calculate the combined time... AB / (A + B) = T Well represent Jenny's time using "Jen." John's time is half of Jenny's, so we can call it ".5Jen." Statement 1 -- SUFFICIENT John takes 3 hours to write the program, half of Jenny's time. So J...

by jjk

Mon Aug 17, 2009 3:10 pm
Forum: Data Sufficiency
Topic: Work problem
Replies: 5
Views: 1422

Stuck between B and E, but will go for B. The policy discussed here is the quarantine of imported domesticated animals. It is indeed an effective policy, since it helps keep Britain rabies-free. The objection one might have to this policy is that it does not control wild animals. That is however ou...

by jjk

Thu Aug 13, 2009 10:31 pm
Forum: Critical Reasoning
Topic: Rabies
Replies: 6
Views: 1527

Chose A. Got kinda nervous seeing everyone else chime in with different answers.

by jjk

Thu Aug 13, 2009 10:11 pm
Forum: Critical Reasoning
Topic: Joker
Replies: 16
Views: 3521
by jjk

Thu Aug 13, 2009 12:04 pm
Forum: Critical Reasoning
Topic: CARS AND DRUGS
Replies: 7
Views: 1988